Question

Let S_n be the collection of permutations on {1,2,...,n}. Consider the cycle s=(1,2,...,n) and consider the...

Let S_n be the collection of permutations on {1,2,...,n}. Consider the cycle s=(1,2,...,n) and consider the cyclic group generate by s, denoted <s>. Prove that the set all t in S_n such that ts=st, is just the set <s>

Homework Answers

Know the answer?
Your Answer:

Post as a guest

Your Name:

What's your source?

Earn Coins

Coins can be redeemed for fabulous gifts.

Not the answer you're looking for?
Ask your own homework help question
Similar Questions
Suppose that the set An has all the even permutations in n-permutations. Prove that this set...
Suppose that the set An has all the even permutations in n-permutations. Prove that this set is the same as [a set consisting of cyclic permutations of length 3 and their products].
Let n ≥ 2. Show that exactly half of the permutations in Sn are even ,...
Let n ≥ 2. Show that exactly half of the permutations in Sn are even , by finding a bijection from the set of all even permutations in Sn to the set of all odd permutations in Sn.
Let T be a linear operator such that T=D+N is a jordan decomposition of T. That...
Let T be a linear operator such that T=D+N is a jordan decomposition of T. That is, D is diagonalizable and N is Nilpotent. Show if S is an endomorphism of V, that is, if S is a linear operator with domain and codmain V, prove if T commutes with S, (ST=TS), then D commutes with S (DS=SD), and N commutes with S (NS=SN).
Let S = {(a1,a2,...,an)|n ≥ 1,ai ∈ Z≥0 for i = 1,2,...,n,an ̸= 0}. So S...
Let S = {(a1,a2,...,an)|n ≥ 1,ai ∈ Z≥0 for i = 1,2,...,n,an ̸= 0}. So S is the set of all finite ordered n-tuples of nonnegative integers where the last coordinate is not 0. Find a bijection from S to Z+.
Prove Theorem 29.10. Let n ∈ Z+. If Ai is countable for all i = 1,2,...,n,...
Prove Theorem 29.10. Let n ∈ Z+. If Ai is countable for all i = 1,2,...,n, then A1 ×A2 ×···×An is countable.
Suppose n and m are integers. Let H = {sm+tn|s ∈ Z and t ∈ Z}....
Suppose n and m are integers. Let H = {sm+tn|s ∈ Z and t ∈ Z}. Prove that H is a cyclic subgroup of Z. ...................... Please help with clear steps that H is a cyclic subgroup of Z
Let U=​{1,2, 3,​ ...,3200​}. Let S be the subset of the numbers in U that are...
Let U=​{1,2, 3,​ ...,3200​}. Let S be the subset of the numbers in U that are multiples of 4​, and let T be the subset of U that are multiples of 9. Since 3200 divided by 4 equals it follows that n(S)=n({4*1,4*2,...,4*800})=800 ​(a) Find​ n(T) using a method similar to the one that showed that n(S)=800 ​(b) Find n(S∩T). ​(c) Label the number of elements in each region of a​ two-loop Venn diagram with the universe U and subsets S...
Let S be a collection of subsets of [n] such that any two subsets in S...
Let S be a collection of subsets of [n] such that any two subsets in S have a non-empty intersection. Show that |S| ≤ 2^(n−1).
Let a be an element of order n in a group and d = gcd(n,k) where...
Let a be an element of order n in a group and d = gcd(n,k) where k is a positive integer. a) Prove that <a^k> = <a^d> b) Prove that |a^k| = n/d c) Use the parts you proved above to find all the cyclic subgroups and their orders when |a| = 100.
Let S be the set {(-1)^n +1 - (1/n): all n are natural numbers}. 1. find...
Let S be the set {(-1)^n +1 - (1/n): all n are natural numbers}. 1. find the infimum and the supremum of S, and prove that these are indeed the infimum and supremum. 2. find all the boundary points of the set S. Prove that each of these numbers is a boundary point. 3. Is the set S closed? Compact? give reasons. 4. Complete the sentence: Any nonempty compact set has a....
ADVERTISEMENT
Need Online Homework Help?

Get Answers For Free
Most questions answered within 1 hours.

Ask a Question
ADVERTISEMENT